QFT: Solving the integral for the Wightman function in Minkowski spacetime.

I'll mark this thread as solved.In summary, the Wightman function for a massless quantum scalar field in 4D Minkowski spacetime can be solved by using contour integration for the k_0 term and then applying a 3D Fourier transform in spherical coordinates for the k_i terms. This results in an integral that can be evaluated using the Residue theorem, providing the desired equation of \langle \hat{\phi}(x) \hat{\phi}(y) \rangle = \frac{1}{4 \pi^2 (x-y)^2}.
  • #1
Aimless
128
0

Homework Statement



How does one actually solve the integral for the Wightman function for a massless quantum scalar field in 4D Minkowski spacetime? That is, what is the integration technique to go from:

[itex] \langle \hat{\phi}(x) \hat{\phi}(y) \rangle = \int_c d^4k \, \frac{1}{(2 \pi )^4}\frac{e^{ik(x-y)}}{ k^2} [/itex]

to:

[itex] \langle \hat{\phi}(x) \hat{\phi}(y) \rangle = \frac{1}{4 \pi^2 (x-y)^2} [/itex]?

Homework Equations



See above.

The Attempt at a Solution



The [itex]k_0[/itex] term is dealt with by contour integration. That part's easy. The problem I'm having is figuring out how to deal with the [itex]k_i[/itex] terms. It should just be a 3D Fourier transform, but I can't figure it out.

I have to admit that it's embarrassing for me to post this, since I'm a postdoc and should know this off the top of my head. I know I have the answer buried somewhere in my notes, but I just finished an oversees move and all of my notes are still in transit. I haven't been able to find it online, either; seems most online lecture notes I've found either just show the final equation or leave the derivation as an exercise to the student. (Ha!)
 
Last edited:
Physics news on Phys.org
  • #2
Aimless said:
The [itex]k_0[/itex] term is dealt with by contour integration. That part's easy. The problem I'm having is figuring out how to deal with the [itex]k_i[/itex] terms. It should just be a 3D Fourier transform, but I can't figure it out.

Can you show your [itex]k_0[/itex] integration? I may have some ideas for the Fourier integral, but first I want to make sure we are talking about the same integral.
 
  • #3
If you consider the [itex]k_0[/itex] integral first, then you get:

[itex]\int_c d^4k \, \frac{i}{(2\pi)^4}\frac{e^{ik(x-y)}}{k^2} = \int_c dk_0 \, \int_{-\infty}^{\infty} d^3\vec{k} \, \frac{i}{(2\pi)^4}\frac{e^{i\vec{k}(\vec{x}-\vec{y})-i k_0 (t_x-t_y)}}{-k_0^2+\vec{k}^2}[/itex]

where the contour for [itex]G^+(x,y)[/itex] is a counterclockwise circle of radius epsilon around the positive root [itex]k_0 = |\vec{k}|[/itex]. (Note that I was missing a factor of [itex]i[/itex] in my first post.)

The Laurent series for [itex]\frac{e^{iux}}{-u^2+a^2}[/itex] around the pole at [itex]u=a[/itex] is [itex]\frac{e^{iax}}{2a(-u+a)}+O(1)[/itex], so by method of residues the [itex]k_0[/itex] integral evaluates to:

[itex]\int_c dk_0 \, \int_{-\infty}^{\infty} d^3\vec{k} \, \frac{i}{(2\pi)^4}\frac{e^{i\vec{k}(\vec{x}-\vec{y})-i k_0 (t_x-t_y)}}{-k_0^2+\vec{k}^2} = \int_{-\infty}^{\infty} d^3\vec{k} \, \frac{1}{(2\pi)^3}\frac{e^{i\vec{k}(\vec{x}-\vec{y})-i \omega_k (t_x-t_y)}}{2 \omega_k} [/itex]

with [itex]\omega_k = |\vec{k}|[/itex].

This is the same integral one gets from evaluation of [itex]\langle \hat{\phi}(x) \hat{\phi}(y) \rangle[/itex] with [itex]\hat{\phi}(x) \equiv \int \frac{d^3\vec{k}}{(2\pi)^{3/2}\sqrt{2\omega_k}} (\hat{a}_\vec{k} e^{i\vec{k}\vec{x}-i\omega_kt}+\hat{a}^\dagger_\vec{k} e^{-i\vec{k}\vec{x}+i\omega_kt}) [/itex], so I don't think it's helpful to evaluate the [itex]k_0[/itex] integral first. I'm not sure, though. The Fourier transform seems more difficult this way, but maybe I'm just not seeing the trick.
 
Last edited:
  • #4
Right, so let's look at the Fourier transform first. It is sufficient to look at [itex]\int_{ \text{all } \vec{k} \text{-space} } d^3 \vec{k} \frac{e^{i\vec{k} \cdot (\vec{x} - \vec{y})}}{\omega_k^2-k_0^2} [/itex] , yes?

I think this is best done in Spherical coordinates [itex](\omega_k, \theta, \phi)[/itex] with the polar axis aligned with [itex](\vec{x} - \vec{y})[/itex], which gives you something like

[tex]2\pi \int_0^{\infty} \int_0^{\pi} \frac{e^{i \omega_k |\vec{x} - \vec{y}| \cos \theta}}{\omega_k^2-k_0^2}\omega_k^2\sin\theta d \omega_k d\theta[/tex]

Do the angular integral first, and then I think you can just use the Residue theorem on the [itex]\omega_k[/itex] integral
 
  • #5
Ah ha! I think you've got it; this seems to stir some memories from my coursework. Let me sit down with pencil and paper and work through it in detail, then I'll post the full derivation.

I'm surprised I didn't come up with that myself. Guess I got stuck on doing the integral in Cartesian coords.
 
  • #6
So, spherical coords. is indeed the right way to solve the integral, although it turns out to be easier to solve if you do the contour first. You get:

[itex]
\langle \hat{\phi}(x) \hat{\phi}(y) \rangle = \int_{-\infty}^{\infty} d^3\vec{k} \, \frac{1}{(2\pi)^3}\frac{e^{i\vec{k}(\vec{x}-\vec{y})-i \omega_k (t_x-t_y)}}{2 \omega_k} \\
= \frac{1}{2(2\pi)^3} \int_0^\infty dk_r \int_{-1}^1 d\cos{k_\theta} \int_0^{2\pi} dk_\phi \, k_r e^{i k_r |\vec{x}-\vec{y}|\cos{k_\theta} - i k_r (t_x-t_y)} \\
= -\frac{1}{4\pi^2} \frac{1}{|\vec{x}-\vec{y}|} \int_0^\infty dk_r \, \sin(k_r |\vec{x}-\vec{y}|)e^{- i k_r (t_x-t_y)}
[/itex]

To evaluate the final integral, you have to displace [itex](t_x-t_y)[/itex] slightly into the complex plane, [itex](t_x-t_y) \to (t_x-t_y -i\epsilon)[/itex], giving:
[itex]-\frac{1}{4\pi^2} \frac{1}{|\vec{x}-\vec{y}|} \int_0^\infty dk_r \, \sin(k_r |\vec{x}-\vec{y}|)e^{- i k_r (t_x-t_y-i\epsilon)} = -\frac{1}{4\pi^2}\frac{1}{|\vec{x}-\vec{y}|^2 - (t_x-t_y)^2} = -\frac{1}{4\pi^2}\frac{1}{(x-y)^2-i\epsilon}[/itex]

This is still a slightly unsatisfying way to do things, since it relies on the identity [itex]\vec{k} \cdot (\vec{x}-\vec{y}) = |\vec{k}||\vec{x}-\vec{y}|\cos \theta[/itex], and that's not guaranteed to hold if I move away from Minkowski spacetime. Oh well; no one ever said QFT was easy.

Thanks for the help, gabba.
 
Last edited:

Related to QFT: Solving the integral for the Wightman function in Minkowski spacetime.

1. What is QFT?

QFT, or Quantum Field Theory, is a theoretical framework in physics that combines the principles of quantum mechanics and special relativity to describe the behavior of subatomic particles and their interactions.

2. What is the Wightman function?

The Wightman function is a mathematical function used in QFT to describe the correlation between different points in Minkowski spacetime. It is a fundamental tool for understanding particle interactions and calculating physical observables.

3. Why is it important to solve the integral for the Wightman function?

Solving the integral for the Wightman function allows us to calculate the correlation between different points in Minkowski spacetime, which is essential for understanding the behavior of particles in QFT. It also helps us to make predictions and test the validity of the theory through experimental results.

4. What is Minkowski spacetime?

Minkowski spacetime is a mathematical model of the universe that combines the three dimensions of space and one dimension of time into a four-dimensional continuum. It is a fundamental concept in special relativity and is used in QFT to describe the behavior of particles.

5. How is the integral for the Wightman function solved?

The integral for the Wightman function is solved using various mathematical techniques, such as perturbation theory, Feynman diagrams, and path integrals. These methods involve breaking down the problem into smaller, solvable parts and using mathematical tools to find a solution that accurately describes the correlation between different points in Minkowski spacetime.

Similar threads

  • Advanced Physics Homework Help
Replies
10
Views
637
Replies
1
Views
840
  • Advanced Physics Homework Help
Replies
2
Views
1K
  • Advanced Physics Homework Help
Replies
1
Views
1K
  • Advanced Physics Homework Help
Replies
1
Views
853
  • Advanced Physics Homework Help
Replies
9
Views
1K
  • Advanced Physics Homework Help
2
Replies
59
Views
7K
Replies
16
Views
623
Replies
27
Views
2K
  • Advanced Physics Homework Help
Replies
1
Views
771
Back
Top